Tài liệu bổ trợ Ngữ pháp và Bài tập Tiếng Anh 12 Sách Mới - Chương trình cả năm - Năm học 2022-2023

pdf 169 trang doantrang27 07/07/2023 1570
Bạn đang xem 20 trang mẫu của tài liệu "Tài liệu bổ trợ Ngữ pháp và Bài tập Tiếng Anh 12 Sách Mới - Chương trình cả năm - Năm học 2022-2023", để tải tài liệu gốc về máy bạn click vào nút DOWNLOAD ở trên

Tài liệu đính kèm:

  • pdftai_lieu_bo_tro_ngu_phap_va_bai_tap_tieng_anh_12_sach_moi_ch.pdf

Nội dung text: Tài liệu bổ trợ Ngữ pháp và Bài tập Tiếng Anh 12 Sách Mới - Chương trình cả năm - Năm học 2022-2023

  1. D. Prior to their advent, there were already several competitors in the field. Question 34. According to paragraph 3, what is the attitude of Robert Bell towards the customers? A. He sees them as modern slaves to consumerism. B. He maintains a healthy relationship of give and take. C. He displays worship in its purest form towards them. D. He views them as the golden goose for his business. Question 35. What does the phrase “stepping stone” in paragraph 4 mean? A. An asset or possession prized as being the best of a group of similar things. B. A person who travels without settling down for any significant period of time C. An important clue to understanding something that is challenging or puzzling. D. An action or event that helps one to make progress towards a specified goal. Read the following passage and mark the letter A, B, C, or D on your answer sheet to indicate the correct answer to each of the questions. Every summer, when the results of university entrance exam come out, many newspaper stories are published about students who are top-scorers across the country. Most portray students as hard-working, studious, smart and, generally, from low-income families. They are often considered heroes or heroines by their families, communes, villages and communities. And they symbolize the efforts made to lift them, and their relatives, out of poverty. The students are often too poor to attend any extra-classes, which make their achievements more illustrious and more newsworthy. While everyone should applaud the students for their admirable efforts, putting too much emphasis on success generates some difficult questions. If other students look up to them as models, of course it's great. However, in a way, it contributes to society's attitude that getting into university is the only way to succeed. For those who fail, their lives are over. It should be noted that about 1.3 million high school students take part in the annual university entrance exams and only about 300,000 of them pass. What's about the hundreds of thousands who fail? Should we demand more stories about those who fail the exam but succeed in life or about those who quit university education at some level and do something else unconventional? "I personally think that it's not about you scoring top in an entrance exam or get even into Harvard. It's about what you do for the rest of your life," said Tran Nguyen Le Van, 29. He is the founder of a website, vexere.com, that passengers can use to book bus tickets online and receive tickets via SMS. His business also arranges online tickets via mobile phones and email. Van dropped out of his MBA at the Thunderbird School of Global Management in Arizona in the United States. His story has caught the attention of many newspapers and he believes more coverage should be given to the youngsters who can be role-models in the start-up community. Getting into university, even with honours, is just the beginning. "We applaud them and their efforts and obviously that can give them motivation to do better in life. However, success requires more than just scores," Van said. Van once told a newspaper that his inspiration also came from among the world's most famous drop-outs, such as Mark Zuckerberg of Facebook or Bill Gates who also dropped out of Harvard University. Alarming statistics about unemployment continues to plague us. As many as 162,000 people with some kind of degree cannot find work, according to Labour Ministry's statistics this month. An emphasis on getting into university does not inspire students who want to try alternative options. At the same time, the Ministry of Education and Training is still pondering on how to reform our exam system, which emphasises theories, but offers little to develop critical thinking or practice. Vu Thi Phuong Anh, former head of the Centre for Education Testing and Quality Assessment at Viet Nam National University in HCM City said the media should also monitor student successes after graduation. She agreed there were many success stories about young people, but added that it was imbalanced if students taking unconventional paths were not also encouraged. P 140
  2. Viet Nam is, more than ever, in desperate need of those who think outside the box. Time for us to recognise talent, no matter where it comes from or how. (Source: Question 36. Which of the following best describes the main idea of this passage? A. Many students consider universities as their only way after school. B. A good model of being successful in the real life out of school. C. Not many students are successful after graduating from universities. D. University is not the only way to success. Question 37. What is NOT stated in the passage about the top-scorers in the entrance exam? A. The majority of them are poor but intelligent and eager to learn. B. They are hoped to find the way to better their families’ lives. C. Their success is more glorious because they attend more classes than others. D. The students are admired for the great efforts. Question 38. The word “unconventional” in paragraph 2 could best be replaced by ___. A. common B. unusual C. well-known D. infamous Question 39. The author described Tran Nguyen Le Van in the third passage as ___. A. a good example to achieve success although he didn’t finish his education. B. a businessman who gains money by selling mobile phones online. C. a founder whose website was inspired from social networks like Facebook. D. a top-scorer who books online tickets and confirm through messages. Question 40. The word “them” in paragraph 3 refers to ___. A. honours B. role-models C. the youngsters D. newspapers Question 41. The word “plague” in paragraph 4 is closest in meaning to ___. A. conflict B. afflict C. remind D. bother Question 42. According to the fourth paragraph, what is TRUE about the modern exam system? A. It puts too much pressure on students who must get a place in a university. B. Students are not encouraged to do something different. C. The government is trying to change the theories of exam. D. Many stories about successful students cannot inspire those who attend universities. Mark the letter A, B, C, or D on your answer sheet to indicate the underlined part that needs correction in each of the following questions. Question 43: A large number of entries has updated in the latest edition of the encyclopedia recently. A. large B. has updated C. of D. recently Question 44: Many places of history, scientific, cultural, or scenic importance have been designated national monuments. A. history B. cultural C. have D. been designated Question 45: Just as the performance ended, all the concertgoers raised to their feet and applauded. A. Just as B. ended C. raised D. their feet Mark the letter A, B, C, or D on your answer sheet to indicate the sentence that is closest in meaning to each of the following questions Question 46: Helen’s boss earns more money than she does A. Helen doesn’t earn as much money as her boss B. Helen’s boss earns less money than she does C. Helen earns more money than her boss D. Helen’s boss earns as much as she does Question 47: Cindy said that “I haven’t seen John since last month.” A. Cindy said she hasn’t seen John since last month B. Cindy said she hadn’t seen John since the previous month P 141
  3. C. Cindy said she wasn’t seen John since the previous month D. Cindy said I hadn’t seen John since the previous month Question 48: John is not here, perhaps he is ill. A. John needn't be here because he is ill. B. Because of his illness, John shouldn't have been here. C. John might be ill, so he is not here. D. John must be ill, so he is not here. Mark the letter A, B, C, or D on your answer sheet to indicate the sentence that best combines each pair of sentences in the following questions Question 49: You can go out with your friends. Make sure you finish your homework by 5 p.m A. As long as you finish your homework by 5 p.m, you can go out with your friends. B. Unless you finish your homework by 5 p.m, you can go out with your friends C. You cannot go out with your friends provided you finish your homework by 5 p.m D. You can’t go out with your friends in case you cannot finish your homework by 5 p.m Question 50. He has great intelligence. He can solve all the problems very quicky A. So intelligent is he that he can solve all the problems very quicky. B. He is very intelligent that he can solve all the problems very quicky. C. An intelligent student is he that he can solve all the problems very quicky D. So intelligent a student is he that he can solve all the problems very quicky. Number 24: Mark the letter A, B, C, or D on your answer sheet to indicate the word whose underlined part differs from the other three in pronunciation in each of the following questions. Question 1: A. changes B. increases C. suffocates D. compromises Question 2: A. hole B. home C. come D. hold Mark the letter A, B, C, or D on your answer sheet to indicate the word that differs from the other three in the position of primary stress in each of the following questions. Question 3: A. report B. conclude C. deter D. sanction Question 4: A. advertise B. definite C. composite D. confusion Mark the letter A, B, C, or D on your answer sheet to indicate the correct answer to each of the following questions. Question 5: Her parents are really strict. They rarely let her stay out late, ___? A. do they B. don’t they C. does she D. doesn’t she Question 6: Dana finally admitted___by what her father had said the day before. A. hurt B. hurting C. being hurt D. hurted Question 7: If she ___ car, she would go out in the evening. A. have B. has C. has had D. had Question 8: When she came home from school yesterday, her mother ___in the kitchen. A. cooked B. was cooking C. is cooking D. cooks Question 9: I will stand here and wait for you you come back. A. because B. though C. so D. until Question 10: ___, he went straight home. A. While he would finish his work B. When he has finished his work C. After he had finished his work D. Before he has been finishing his work Question 11: Keep children___from the fire. A. in with B. from C. away D. back Question 12: The scientists___on this project try to find the solutions to air pollution. A. working B. worked C. are working D. who working Question 13: His___of the generator is very famous. A. invent B. inventive C. invention D. inventor Question 14: The government hopes to___its plans for introducing cable TV. A. turn out B. carry out C. carry on D. keep on P 142
  4. Question 15: The jury___her compliments on her excellent knowledge of the subject. A. paid B. gave C. made D. said Question 16: We took the children to the ___park last weekend. They really enjoyed going on all the rides. A. wildlife B. amusement C. national D. entertainment Question 17: Nobody took any___of the warning and they went swimming in the contaminated water. A. information B. attention C. sight D. notice Question 18: Tony was clearly nervous; he was sitting right on the___ of his chair. A. outside B. edge C. tip D. border Question 19: As I was walking along the street, I saw a $10 note on pavement. A. a B. an C. the D. no article Mark the letter A, B, C, or D on your answer sheet to indicate the word(s) CLOSEST in meaning to the underlined word(s) in each of the following questions. Question 20: The sign indicates the correct direction to the store A. looks B. shows C. says D. needs Question 21: There was always sufficient money to go out for dinner A. not enough B. too much C. enough D. none Mark the letter A, B, C, or D on your answer sheet to indicate the word(s) OPPOSITE in meaning to the underlined word(s) in each of the following questions. Question 22: His career in the illicit drug trade ended with the police raid this morning. A. elicited B. irregular C. secret D. legal Question 23: Though I persuaded my boss to solve a very serious problem in the new management system, he just made light of it. A. completely ignored B. treated as important C. disagreed with D. discovered by chance Mark the letter A, B, C, or D on your answer sheet to indicate the sentence that best completes each of the following exchanges. Question 24: “What an attractive hair style you have got, Mary!” - “___” A. Thank you very much. I am afraid B. I don't like your sayings C. You are telling a lie D. Thank you for your compliment Question 25: Two friends Diana and Anne are talking to each other about their shopping. Diana: “Look at this catalog, Anne. I think I want to get this red blouse.” Anne: “___” A. Don’t you have one like this in blue? B. That’s a long way to go, dear. C. No, thank you. D. I’ll go myself, then. Read the following passage and mark the letter A, B, C, or D on your answer sheet to indicate the correct word or phrase that best fits each of the numbered blanks from 26 to 30. In the past, the process of choosing a career was a much simpler matter than it is today. Aboy often followed in his father’s footsteps. His sister learned the household skills that would prepare her to become (26)___ wife and mother. Nowadays young people grow up in a much freer society (27)___they enjoy almost unlimited career opportunities. In recent years, there (28)___an enormous increase in the kinds of vocations from which it is possible to choose. In addition, many of the barriers to career opportunity that existed only a few decades ago, such as (29)___based on sex or religion or ethnic origins, are (30) disappearing. Question 26: A. many B. the C. an D. a Question 27: A. where B. when C. why D. whom Question 28: A. had been B. has been C. will be D. was Question 29: A. judgement B. perception C. devotion D. discrimination Question 30: A. rapidly B. incessantly C. categorically D. vigilantly P 143
  5. Read the following passage and mark the letter A, B, C, or D on your answer sheet to indicate the correct answer to each of the questions from 31 to 35. After twenty years of growing student enrollments and economic prosperity, business schools in the United States have started to face harder times. Only Harvard’s MBA School has shown a substantial increase in enrollment in recent years. Both Princeton and Stanford have seen decreases in their enrollments. Since 1990, the number of people receiving Masters in Business Administration (MBA) degrees, has dropped about 3 percent to 75,000, and the trend of lower enrollment rates is expected to continue. There are two factors causing this decrease in students seeking an MBA degree. The first one is that many graduates of four-year colleges are finding that an MBA degree does not guarantee a plush job on Wall Street, or in other financial districts of major American cities. Many of the entry- level management jobs are going to students graduating with Master of Arts degrees in English and the humanities as well as those holding MBA degrees. Students have asked the question, “Is an MBA degree really what I need to be best prepared for getting a good job?” The second major factor has been the cutting of American payrolls and the lower number of entry- level jobs being offered. Business needs are changing, and MBA schools are struggling to meet the new demands. Question 31. What is the main focus of this passage? A. Jobs on Wall Street B. Types of graduate degrees C. Changes in enrollment for MBA schools D. How schools are changing to reflect the economy Question 32. The word “prosperity” in the first paragraph could be best replaced by which of the following? A. success B. surplus C. nurturing D. education Question 33. Which of the following business schools has shown an increase in enrollment? A. Princeton B. Harvard C. Stanford D. Yale Question 34. Which of the following descriptions most likely applies to Wall Street? A. a center for international affairs B. a major financial center C. a shopping district D. a neighborhood in New York Question 35: As used in the second paragraph, the word “struggling” is closest in meaning to___. A. evolving B. plunging C. starting D. striving Read the following passage and mark the letter A, B, C, or D on your answer sheet to indicate the correct answer to each of the questions from 36 to 42. According to sociologists, there are several different ways in which a person may become recognized as the leader of a social group in the United States. In the tamily, traditional cultural patterns confer leadership on one or both of the parents. In other cases, such as friendship groups, one or more persons may gradually emerge as leaders, although there is no formal process of selection. In larger groups, leaders are usually chosen formally through election or recruitment. Although leaders are often thought to be people with unusual personal ubility, decades ot research have failed to produce consistent evidence that there is any category ot “natural leaders”. It seems that there is no set of personal qualities that all leaders have in common; rather, virtually any person may be recognized as a leader if the person has qualities that meet the needs ot that particular group. Furthermore, although it is commonly supposed that social groups have a single leader, research suggests that there are typically two different leadership roles that are held by different individuals. Instrumental leadership is leadership that emphasizes the completion of tasks by a social group. Group members look to instrumental leaders to “get things” done.” Expressive leadership, on the other hand, is leadership that emphasizes the collective well-being of a social group’s member. Expressive leaders are less concerned with the overall goals of the group than with providing P 144
  6. emotional support to group members and attempting to minimize tension and conflict among them. Group members expect expressive leaders to maintain stable relationships within the group and provide support to individual members. Instrumental leaders are likely to have a rather secondary relationship to other group members. They give orders and may discipline group members who inhibit attainment of the group’s goals. Expressive leaders cultivate a more personal or primary relationship to others in the group. They offer sympathy when someone experiences difficulties or is subjected to discipline, are quick to lighten a serious moment with humor, and try to resolve issues that threaten to divide the group. As the differences in these two roles suggest, expressive leaders generally receive more personal affection from group members; instrumental leaders, if they are successful in promoting group goals, may enjoy a more distant respect. Question 36. What does the passage mainly discuss? A. The problems faced by leaders B. How leadership differs in small and large groups C. How social groups determine who will lead them D. The role of leaders in social groups Question 37. The passage mentions all of the following ways by which people can become leaders EXCEPT___. A. recruitment B. formal election process C. specific leadership training D. traditional cultural patterns Question 38. In mentioning “natural leaders” in the second paragraph, the author is making the point that___. A. few people qualify as “natural leaders” B. there is no proof that “natural leaders” exist C. “natural leaders’ are easily accepted by the members of a social group D. “natural leaders” share a similar set of characteristics Question 39. The passage indicates that ‘instrumental leaders’ generally focus on . A. ensuring harmonious relationships B. sharing responsibility with group members C. identifying new leaders D. achieving a goal Question 40. The word “collective” in the third paragraph is closest in meaning to . A. necessary B. typical C. group D. particular Question 41. The word “them” in the third paragraph refers to . A. expressive leaders B. goals of the group C. group members D. tension and conflict Question 42. A “secondary relationship” mentioned in the last paragraph between a leader and the members of a group could best be characterized as . A. distant B. enthusiastic C. unreliable D. personal Mark the letter A, B, C, or D on your answer sheet to indicate the underlined part that needs correction in each of the following questions. Question 43: A number of tourists (A) is going to (B) return the evaluation form distributed (C) by the travel agent (D) . Question 44: After teaching (A) English in Vietnam for (B) one year, Philip decided to buy (C) a house and spending (D) the rest of his life there. Question 45: Our neighbor (A) is quite safe (B) because there have not been many (C) crimes recently (D). Mark the letter A, B, C, or D on your answer sheet to indicate the sentence that is closest in meaning to each of the following questions. Question 46: A supermarket is more convenient than a shopping centre. A. A shopping centre is not as convenient as a supermarket. B. A shopping centre is more convenient than a supermarket. C. A supermarket is not as convenient as a shopping centre. D. A supermarket is as inconvenient as a shopping centre. Question 47: "It was your fault. You broke my windows, "said the woman to him. A. The woman insisted him on breaking her windows. P 145
  7. B. The woman advised him to break her windows. C. The woman told him to break her windows. D. The woman blamed him for having broken her windows. Question 48. It was a mistake for Tony to buy that house. A. Tony couldn’t have bought that house. B. Tony can’t have bought that house. C. Tony needn’t have bought that house. D. Tony shouldn’t have bought that house. Mark the letter A, B, C, or D on your answer sheet to indicate the sentence that best combines each pair of sentences in the following questions. Question 49: They were late for the meeting. The heavy was heavy. A. If it snowed heavily, they would be late for the meeting. B. Had it not snowed heavily, they would have been late for the meeting. C. But for the heavy snow, they wouldn't have been late for the meeting. D. If it didn't snow heavily, they wouldn't be late for the meeting. Question 50: Seth informed us of his retirement from the company. He did it when arriving at the meeting. A. Only after his retiring from the company did Seth tell us about his arrival at the meeting. B. Not until Seth said to us that he would leave the company did he turn up at the meeting. C. Hardly had Seth notified us of his retiring from the company when he arrived at the meeting. D. No sooner had Seth arrived at the meeting than we were told about his leaving the company. Number 26: Mark the letter A, B, C, or D to indicate the word whose underlined part differs from the other three in pronunciation in each of the following questions. Question 1: A. serves B. hopes C. likes D. writes Question 2: A. teacher B. clean C. great D. means Mark the letter A, B, C, or D to indicate the word that differs from the other three in the position of the primary stress in each of the following questions. Question 3: A. mistake B. unite C. wonder D. behave Question 4: A. persistent B. dynamic C. sensitive D. ambitious Mark the letter A, B, C or D to indicate the correct answer to each of the following questions. Question 5: “Give me another chance, ___?” A. don’t you B. aren’t you C. shall you D. will you Question 6: The old woman still recalls clearly ___ by her teacher when she was late on her first day at school. A. to be criticised B. to have criticised C. being criticised D. criticising Question 7: If Jim hadn’t tried to kill that millionaire, he ___ in prison today. A. hadn’t been B. won’t be C. wouldn’t be D. wouldn’t have been Question 8: I haven’t met Sally since we ___ school. A. left B. had left C. would leave D. was leaving Question 9: He is a very intelligent boy; ___, he sometimes gets bad marks. A. otherwise B. thus C. so D. however Question 10: ___, we had already put out the fire. A. Until the firemen arrived to help B. No sooner the firemen arrived to help C. By the time the firemen arrived to help D. After the firemen arrived to help Question 11: I believe that judges should be independent ___ the government. A. to B. of C. with D. on Question 12: He is only one boy___ in this game. A. who participating B. participated C. to participate D. who participate. Question 13: Universities send letters of ___ to successful candidates by post. A. accept B. acceptable C. acceptably D. acceptance P 146
  8. Question 14: My parents are busy workers and I often ___ my younger brother after school. A. take after B. take care of C. try out D. look up Question 15: At the end of the film, the young prince ___ in love with a reporter. A. felt B. made C. fell D. got Question 16: Early to bed and early to rise will ___ you good. A. make B. bring C. do D. help Question 17: The first week of classes at university was a little ___because so many students get lost, change classes or go to the wrong place. A. disarranged B. chaotic C. uncontrolled D. famous Question 18: The captain has not decided yet where to stop on the journey – we’ll just play it by___ and see how we feel. A. mouth B. ear C. eye D. hand Question 19: I have left my book in ___ kitchen and I would like you to get it for me. A. the B. a C. ỉ D. an Mark the letter A, B, C or D to indicate the word(s) CLOSEST in meaning to the underlined word(s) in each of the following questions. Question 20: The protesters were angry with the council’s plan to do away with a lovely old building and put a car park there instead. A. destroy B. replace C. remain D. keep Question 21: There are many TV commercials which distracting viewers from watching their favorite films. A. economics B. businesses C. contests D. advertisements Mark the letter A, B, C or D to indicate the word(s) OPPOSITE in meaning to the underlined word(s) in each of the following questions. Question 22: Many people perished in the Kobe earthquake because they were not prepared for it. A. survived B. departed C. lost their lives D. declined Question 23: The writer was really hot under the collar when his novel was mistaken for another. A. angry B. worried C. calm D. curious Mark the letter A, B, C, or D on your answer sheet to indicate the sentence that best completes each of the following exchanges Question 24: Mai and Lan are friends.Lan asks Mai about Mai's plan. Select the most suitable response to fill in the blank. Lan: “Are you going to see the live show by Son Tung today?” Mai: “___”. A. Yes, I enjoyed it very much B. Maybe I'll be out C. Yes, I'm going to stay in D. I think so Question 25: Mary invited her friend, Sarah, to have dinner out that night and Sarah accepted. Choose the most suitable response to fill in the blank in the following exchange. Mary: “Shall we eat out tonight?” - Sarah: “___.” A. It's kind of you to invite B. You are very welcome C. That's a great idea D. That's acceptable Read the following passage and mark the letter A, B, C, or D on your answer sheet to indicate the correct word or phrase that best fits each of the numbered blanks from 26 to 30. In such a costly and competitive society and world, no one of us can live without money. We need money to fulfill our basic needs of the life such as buying food, and (26) ___ many basic necessities of life which are almost impossible to buy without money. People in the society (27)___ _are rich and have property are looked as honourable and respectful person of the society however a poor person is seen as hatred without any good impression. P 147
  9. Money increases the position of the person in the society and (28)___ _a good impression to him. All of us want to be rich by earning more money through good job or business in order to fulfil all the increasing demands of the modern age. (29)___ _, only few people get this chance of completing their dreams of being a millionaire. So, money is the thing of great importance all through the life. Money is required by everyone whether he/she is rich or poor and living in urban areas or rural areas. People in the urban areas are earning more money than the people living in backward or rural areas as the people of the urban areas have more (30)___ _to the technologies and get more opportunity because of the easy sources. (Adapted from Question 26: A. other B. some C. many D. few Question 27: A. where B. what C. who D. which Question 28: A. gives B. does C. takes D. draws Question 29: A. Besides B.Therefore C. Moreover D. However Question 30: A. way B. exit C. access D. order Read the following passage and mark the letter A, B, C, or D on your answer sheet to indicate the correct answer to each of the questions from 31 to 35. For many American university students, the weeklong spring break holiday means an endless party on a sunny beach in Florida or Mexico. In Panama City Beach, Florida, a city with a permanent population of around 36,000, more than half a million university students arrive during the month of March to play and party, making it the number one spring break destination in the United States. A weeklong drinking binge is not for anyone, however, and a growing number of American university students have found a way to make spring break matter. For them, joining or leading a group of volunteers to travel locally or internationally and work to show problems such as poverty, homelessness, or environmental damage makes spring break a unique learning experience that university students can feel good about. Students who participate in alternative spring break projects find them very rewarding. While most university students have to get their degrees before they can start helping people, student volunteers are able to help people now. On the other hand, the accommodations are far from glamorous. Students often sleep on the floor of a school or spend the week camping in tents. But students only pay around $250 for meals and transportation, which is much less than some of their peers spend to travel to more traditional spring break hotspots. Alternative spring break trips appear to be growing in popularity at universities across the United States. Students cite a number of reason for participating. Some appreciate the opportunity to socialize and meet new friends. Others want to exercise their beliefs about people’s obligation to serve humanity and make the world a better place. Whatever their reason, these students have discovered something that gives them rich rewards along with a break from school work. Question 31. What is the passage mainly about? A. Students’ travelling preferences B. A traditional approach to spring breaks C. American students’ social life D. Students’ alternative spring breaks Question 32. How many university students travel to Panama Beach City every March for spring break? A. Around 10,000 B. Around 36,000 C. Around 500,000 D. Around 50,000 Question 33. The word “cite” in paragraph 2 probably means ___. A. listing B. getting C. avoiding D. inventing Question 34. The word “them” in paragraph 1 refers to ___. A. degrees B. people C. projects D. students Question 35. Which of the following is NOT mentioned as a problem that alternative spring break trips try to help solve? A. Environment damage B. Homelessness C. Poverty D. Overpopulation Read the following passage and mark the letter A, B, C, or D on your answer sheet to indicate the correct answer to each of the following questions from 36 to 42. P 148
  10. What is commonly called pepper in reality comes from two very different families of plants. Black and white pepper both come from the fruit of the Piper nigrum, a vine with fruits called peppercorns. The peppercorns turn from green to red as they ripen and finally blacken as they dry out. The dried-out peppercorns are ground to obtain black pepper. White pepper, which has a more subtle flavour than black pepper, comes from the same peppercorns as black pepper. To obtain white pepper, the outer hull of the peppercorn, the pericarp, is removed before the peppercorn is ground. Red and green pepper, on the other hand, come from a completely different family from black and white pepper. Red and green peppers are from the genus Capsicum. Plants of this type generally have tiny white flowers and fruit which can be any of a number of colours, shapes and sizes. These peppers range in flavour from very mild and sweet to the most incredibly burning taste imaginable. Bell peppers are the most mild, while habanros are the most burning. Christopher Columbus is responsible for the present-day confusion over what pepper is. The Piper nigrum variety of pepper was highly valued for centuries, and high demand for pepper by Europeans was a major cause of the fifteen-century push to locate ocean routes to the spice- growing regions of Asia. When Columbus arrived in the New World in 1492, he was particularly interested in finding black pepper because of the high price it would command in Europe. Columbus came across plants from the Capsicum family in use among people of the New World, and he incorrectly identified them as relatives of black pepper. Columbus introduced the spicy Capsicum chili peppers to Europeans on his return from the 1492 voyage, and traders later spread them to Asia and Africa. These Capsicum peppers have continued to be called peppers in spite of the fact that they are not related to the black and white pepper of the Piper nigrum family. Question 36: The word turn could best be replaced by ___. A. revert B. veer C. exchange D. change Question 37: According to the passage, both black and white peppers ___. A. have the same flavour B. come from different plants C. change colours after they are ground D. are ground from dried-out peppercorns Question 38: What part of the Piper nigrum is the pericarp? A. The seed inside the fruit B. The outer covering of the vine C. The pulp inside the vine D. The outer covering of the fruit Question 39: What usually does NOT vary in a Capsicum plant? A. The size of the fruit B. The colour of the flower C. The colour of the fruit D. The shape of the fruit Question 40: The word push could best be replaced by ___. A. hit B. drive C. shove D. strength Question 41: The pronoun them refers to ___. A. Europeans B. plants C. people D. relatives Question 42: It can be inferred from the passage that chili peppers originally came from ___. A. Europe B. Asia C. America D. Africa Mark the letter A, B, C, or D on your answer sheet to indicate the underlined part that needs correction in each of the following questions. Question 43: Neither his parents nor his teacher were satisfied with his result when he was at high school. A. Neither B. were C. with D. was Question 44: The examination will test your ability to understand spoken English, to read non- technical language and writing language A.will test B. spoken C. non – technical language D. writing Question 45: The sign says that we should read the constructions carefully before proceeding. A. says B. should C. the constructions D. proceeding Mark the letter A, B, C, or D on your answer sheet to indicate the sentence that is closest in meaning to each of the following questions P 149
  11. Question 46: Mai is the most beautiful girl in my class A. Noone in my class is more beautiful than Mai. B. Mai is not as beautiful as anyone in my class. C. Mai is more beautiful than everyone in my class. D. Mai is less beautiful than veryone in my class. Question 47: “Mum, please don’t tell Dad my mistake!” the boy said. A. The boy insisted his mother not tell his father his mistake. B. The boy told his mother not to mention his mistake any more. C. The boy asked his mother not to tell his father his mistake. D. The boy wanted his mother to keep his mistake in her heart. Question 48: You are able to go out with your friend this evening A. You musn’t go out with your friend this evening. B. You should go out with your friend this evening. C. You needn’t go out with your friend this evening. D. You can go out with your friend this evening. Mark the letter A, B, C, or D on your answer sheet to indicate the sentence that best combines each pair of sentences in the following questions Question 49: If it were not for Helen's wonderful acting, the play would be a flop. A. Helen acted so wonderfully, but the play was a flop. B. But for Helen acting so wonderfully, the play would be a flop. C.The play was a flop although Helen acted so wonderfully. D.The play was a flop although Helen was such a wonderful actor. Question 50: Right after the boy got out of his house, it started to rain heavily. A. It had rained heavily before the boy got out of his house. B. No sooner had the boy got out of his house than it started to rain heavily. C. Not until it started to rain heavily did the boy got out of his house. D. Hardly had it started to rain heavily when the boy got out of his house. Number 27: Mark the letter A, B, C, or D to indicate the word whose underlined part differs from the other three in pronunciation in each of the following questions Cõu 1. A. absence B. arrive C. absorb D. apologize Cõu 2. A. dogged B. crooked C. naked D. backed Mark the letter A, B, C, or D to indicate the word that differs from the other three in the position of the primary stress in each of the following questions. Cõu 3. A. approach B. panda C. income D. current Cõu 4. A. confidence B. celebrate C. effective D. handicapped Mark the letter A, B, C or D to indicate the correct answer to each of the following questions. Cõu 5. It’s a lovely day___? A. isn’t it B. was it C. doesn’t it D. did it Cõu 6. Tom hopes ___ a solution soon. A. to find B. finding C. find D. found Cõu 7. Though she lost her job last month, she still wanted to save___ so she said that she had left it willingly. A. mouth B. face C. reputation D. fame Cõu 8. The final winner will be the one who breaks through ___ and survives till the last minutes. A. obstacles B. difficulty C. hindrance D. impediment Cõu 9. Richard will look for a job ___. A. after he had passed his exams B. before he passed his exams C. while he was passing his exams D. as soon as he passes his exams P 150
  12. Cõu 10. Lan has learnt English since she___ a small girl. A. is B. was C. has been D. had been Cõu 11. Organizations may qualify for exemption from income tax if they ___ exclusively as charities. A. operated B. operate C. had operated D. are operated Cõu 12. I used to be nervous when my father asked me to give him my school ___. A. diploma B. certificate C. report D. background Cõu 13. Simon Lake drew the inspiration ___ La submarine of undersea travel and exploration from Twenty Thousand Leagues Under the Sea. A. at B. by C. for D. of Cõu 14. Parents should start teaching them how to manage more complicated duties ___ children graduate from toddlerhood and move toward preschool. A. yet B. when C. and D. so Cõu 15. They are going to ___ the pool to 1.8 meter. A. deepen B. depth C. deep D. deeply Cõu 16. In a formal interview, it is essential to maintain good eye___with the interviewers. A. link B. connection C. touch D. contact Cõu 17. The event received coverage in a newspaper___ in the area. A. was widely distributed B. having distributed widely C. widely distributed D. widely distributing Cõu 18. There are other problems of city life which I don't propose to ___ at the moment. A. go into B. go around C. go for D. go up Cõu 19. My brother has been playing piano since he was a small child. A. the B. a C. no article D. an Mark the letter A, B, C or D to indicate the word(s) CLOSEST in meaning to the underlined word(s) in each of the following questions. Cõu 20. In the 1960s, the World Health Organization (WHO) began a campaign to get rid of the mosquitoes that transmit the disease malaria. A. minimize B. develop C. eradicate D. appreciate Cõu 21. A father will be his child's role model. He will be the example for his child of what husbands and fathers are like. A. someone that others admire and follow B. someone that others share interest with C. someone that others love and live with D. someone that others are fond of with Mark the letter A, B, C or D to indicate the word(s) OPPOSITE in meaning to the underlined word(s) in each of the following questions. Cõu 22. For many couples, money is the source of arguments, frustration. When it comes to finances and relationships, sharing the financial burden is important. A. benefit B. responsibility C. aid D. difficulty Cõu 23. Aren't you putting the cart before the horse by deciding what to wear for the wedding before you've even been invited to it? A. doing things in the wrong order B. do things in the right order C. knowing the ropes D. upsetting the apple cart Read the following passage and mark the letter A, B, C, or D on your answer sheet to indicate the correct word or phrase that best fits each of the numbered blanks. Human beings are extremely diverse in very many ways. People differ (24) ___ opinions, race, nationality, gender, age, sexuality, ethnicity, class, religion, lifestyle and so much more, yet at the very basic we are all human species. Ideally, all people feel (25) ___ and joy despite the P 151
  13. differences. Today, the changes in time and technology have made is extremely impossible for any group of people to live without interacting with others outside their group. Often people of different cultural and geographical (26) ___ meet in international conferences, education exchange programs, sports, etc. Sadly, the history of mankind discriminating against each other based on cultural differences has been with us for ages. So many people have died or have been denied their rights (27) ___ individual greed. Properties and economies have also been destroyed due to lack of understanding. Unfortunately, some of these occurrences are visible even today, perpetuated by people who little understand the importance of the (28)___of our diversity. (Source: Cõu 24. A. from B. by C. in D. with Cõu 25. A. pain B. pained C. painful D. painless Cõu 26. A. backgrounds B. settings C. identities D. surroundings Cõu 27. A. because of B. thanks to C. despite D. as a result Cõu 28. A. specialty B. uniqueness C. identity D. sustainability Mark the letter A, B, C or D to indicate the underlined part that needs correction in each of the following questions. Question 28: Books in the home is (A) a wonderful (B) source of (C) knowledge and pleasure (D) . Question 30: Folklore (A) consists of (B) the beliefs, customs, traditions, and telling stories (C) that people pass from generation to generation (D). Question 31: Economical (A) growth is one of (B) the factors that (C) help a country develop (D) . Read the following passage and mark the letter A, B, C, or D on your answer sheet to indicate the correct answer to each of the questions. The best explainer videos are realized by great background score. When was the last time you watched a silent movie? Never? Not for a while? A long while? Even if you remember watching one, it would still have had a background score, or you’ve accidentally pressed the mute button! The importance of background music and sound effects for your explainer video, therefore, cannot be overstated. You may not need a full blown orchestra or classy jazz tones to create the mood, but whether going for a subtle effect, a professional demonstration or simply trying to attract customers, music is a must. Explainer videos have the emotional quotient working for them and music is just going to add more to make the viewers relate to the pain points you are highlighting in your video. Imagine if your favorite video game has no background music or sound effects to back the amazing graphics? Would you play it with the same feel and excitement? Nah! Same is the case with your explainer videos. They need to and should have apt sound effects to make them worth your viewer’s time. While the focus of explainer videos production is more on passing the desired information and explaining technical and complex procedures easily, it doesn’t mean you’ll be distracting your viewers with the background music. The sound effects and music is not going to take the message away from your explainer, it’s just going to enhance it and turn it more watchable and share-able. Even if your video doesn’t require a background score, it must include some sound effects or else it would be not so good an experience for those who choose to watch it. (source: Cõu 32. Which best serves as the title for the passage? A. The importance of background music and sound effects. B. The influence of silence and background music on memory. C. Inside the booming business of background music. D. How to choose the background music for your explainer videos? Cõu 33. The word “score” in paragraph 1 can be replaced by ___. A. grove B. mark C. instrument D. music P 152
  14. Cõu 34. According to paragraph 2, what is the main purpose for using background music? A. Sound helps build and sustain relationships between films and viewers. B. Background music can control how the audience should react to a scene. C. Background music can influence both everyone’s mood and choices. D. A musical atmosphere can actually motivate people to do risky things. Cõu 35. The word “it” in paragraph 3 refers to ___. A. score B. video C. sound D. background Cõu 36. According to paragraph 3, what is the fundamental and typical use of explainer videos? A. It is used to distill wide-ranging and complex ideas into a viewer-friendly package. B. Businesses use them to quickly introduce themselves and their importance. C. It is designed as a means to stimulate both auditory and visual senses. D. It helps the customers with their daily life problems with scientific information. Read the following passage and mark the letter A, B, C, or D on your answer sheet to indicate the correct answer to each of the questions. In our connected globalized world, the languages which dominate communications and business, Mandarin, Hindi, English, Spanish and Russian amongst others, are placing small languages spoken in remote places under increasing pressure. Fewer and fewer people speak languages such as Liki, Taushiro and Dumi as their children shift away from the language of their ancestors towards languages which promise education, success and the chance of a better life. While to many parents, this may appear a reasonable choice, giving their offspring the opportunity to achieve the sort of prosperity they see on television, the children themselves often lose touch with their roots. However, in many places the more reasonable option of bilingualism, where children learn to speak both a local and a national language, is being promoted. This gives hope that many endangered languages will survive, allowing people to combine their links to local tradition with access to wider world culture. While individuals are free to choose if they wish to speak a minority language, national governments should be under no obligation to provide education in an economically unproductive language, especially in times of budget constraints. It is generally accepted that national languages unite and help to create wealth while minority regional languages divide. Furthermore, governments have a duty to ensure that young people can fulfil their full potential, meaning that state education must provide them with the ability to speak and work in their national language and so equip them to participate responsibly in national affairs. People whose language competence does not extend beyond the use of a regional tongue have limited prospects. This means that while many people may feel a sentimental attachment to their local language, their government’s position should be one of benign neglect, allowing people to speak the language, but not acting to prevent its eventual disappearance. Many PhD students studying minority languages lack the resources to develop their language skills, with the result that they have to rely on interpreters and translators to communicate with speakers of the language they are studying. This has a detrimental effect on the quality of their research. At the same time, they have to struggle against the frequently expressed opinion that minority languages serve no useful purpose and should be allowed to die a natural death. Such a view fails to take into account the fact that a unique body of knowledge and culture, built up over thousands of years, is contained in a language and that language extinction and species extinction are different facets of the same process. They are part of an impending global catastrophe which is beginning to look unavoidable. (Adapted from Complete Advanced by Guy Brook – Hart and Simon Haines) Cõu 37. Which of the following could best serve as the main topic of the passage? A. The threat to minority languages in different parts of the world. B. The domination of business languages all over the world. C. The shift from regional to national languages in many countries. D. The benefits of national languages in modern world. P 153
  15. Cõu 38. According to the first paragraph, why do many parents consider the change towards national languages a reasonable choice? A. Because not many people nowadays are familiar with the language of their ancestors. B. Because children now can learn to speak both a local and a national language. C. Because their children may have a chance to achieve education, success and better living condition. D. Because their children may help to combine their links to local tradition with access to wider world culture. Cõu 39. What does the word “this” in the first paragraph refer to? A. the more reasonable option B. the place where children can use bilingualism C. the opportunity to achieve prosperity D. the promotion of bilingualism Cõu 40. The word “constraints” in the second language is closest in meaning to ___. A. investments B. restrictions C. crises D. depressions Cõu 41. What is the influence of the shortage of minority language resources on many PhD students mentioned in paragraph 3? A. These students are unable to persuade people to use the language they are learning. B. Their qualified research is unlikely to complete. C. They will have many difficulties in understanding these languages. D. They have to become interpreters and translators of these languages themselves. Cõu 42. The word “facets” in the last paragraph could be best replaced by ___. A. aspects B. problems C. procedures D. products Cõu 43. Which statement is NOT true according to the passage? A. Local languages can’t unite and create as much wealth as national languages. B. State education is obligatory to ensure young people to speak and work in their regional languages. C. Lack of language skills causes a negative effect on the quality of PhD students’ research. D. Each citizen has their right to decide whether they want to speak a minority language or not. Mark the letter A, B, C, or D to indicate the sentence that is closest in meaning to each of the following questions. Question 44: I can’t cook as well as my mother can. A. My mother can cook well than I can. B. My mother can cook better than I can. C. My mother can’t cook better than I can. D. I can cook better than my mother can. Question 45: “You’d better apologize for being late,” said my mother. A. My mother suggested apologizing for being late. B. My mother suggested me to apologize for being late. C. My mother advised me to apologize for being late. D. My mother warned me to apologize for being late. .Question 46: You are not allowed to throw litter on the streets. A. You may throw litter on the streets. B. You should throw litter on the streets. C. You mustn’t throw litter on the streets. D. You needn’t throw litter on the streets. Mark the letter A, B, C or D to indicate the option that best completes following exchanges. Question 47: David is talking to Lucy about her pair of trainers. David: “How fashionable a pair of trainers you have!” - Lucy: “___” A. Do you want to know where I bought them? B. I know it’s fashionable. C. Thanks for your compliment. D. Yes, of course. Question 48: Nam and Nga are talking with each other about artificial intelligence. Nam: "I think that the rapid development of artificial intelligence would pose a threat to humankind. What do you think?” Nga: "___. Human beings are very smart. They create artificial intelligence and would know how to keep it under control.” A. No. I don't think so B. I totally agree with you P 154
  16. C. You can't say it again D. What on earth you are talking about Mark the letter A, B, C, or D to indicate the sentence that best combines each pair of sentences in the following questions. Question 49: He helps me every day. I can finish my homework. A. I would not finish my homework if he helped me every day. B. Provided he helped me every day, I would not finish my homework. C. Unless he helps me every day, I will finish my homework. D. But for his daily help, I would not finish my homework. Question 50: He retired. He then thought about having a holiday abroad. A. Having thought about having a holiday abroad, he retired. B. Not until he had retired did he think about having a holiday abroad. C. Had he thought about having a holiday abroad, he wouldn’t retire. D. Only after he thought about having a holiday abroad did he retire. Number 28 Mark the letter A, B, C, or D on your answer sheet to indicate the word whose underlined part differs from the other three in pronunciation in each of the following questions. Question 1. A. sees B. sports C. pools D. trains Question 2. A. sour B. hour C. pour D. flour Mark the letter A, B, C, or D on your answer sheet to indicate the word that differs from the other three in the position of primary stress in each of the following questions. Question 3. A. image B. predict C. envy D. cover Question 4. A. preference B. attraction C. advantage D. infinitive Mark the letter A, B, C, or D on your answer sheet to indicate the correct answer to each of the following questions. Question 5. Due to lacking ___, they couldn’t open a new shop as scheduled. A. interest rates B. expenses C. shares D. funds Question 6. Armed terrorists are reported to have ___ the Embassy. A. taken up B. taken to C. taken over D. taken after Question 7. I think that married couples should be financially independent ___ their parents. A. to B. of C. with D. on Question 8. The more ___ you look, the better you will feel. A. confident B. confide C. confidently D. confidence Question 9. By appearing on the soap powder commercials, she became a name. A. housekeeper B. housewife C. household D. house Question 10. Your sister seldom cooks meals,___? A. did she B. didn’t she C. does she D. doesn’t she Question 11. I must take this watch to be repaired as it ___ over 20 minutes a day. A. accelerates B. increases C. progresses D. gains Question 12. By the time you finish cooking they their homework. A. will have finished B. will finish C. had finished D. have finished Question 13. Tommy is on the way to his friend's birthday party, carrying a gift box ___ colourful paper. A. were nicely wrapped B. having wrapped nicely C. nicely wrapped D. nicely wrapping Question 14. His health has improved a lot since he ___ doing regular exercise. A. starts B. started C. has started D. had started Question 15. ___ they are delicious, hamburgers and French fries are too high in fat. A. However B. Although C. Because D. Despite Question 16. It is advisable that we should avoid leaded petrol to save the environment. A. use B. to use C. using D. be used P 155
  17. Question 17. He may be shy now, but he will soon come out of his ___ when he meets the right girl. A. shoe B. hole C. shed D. shell Question 18. Students will not be allowed into the exam room if they ___ their student cards. A. don’t produce B. didn’t produce C. produced D. hadn’t produced Question 19: ___Socialist Republic of Vietnam is a country with a great patriotism through its history. A. The B. A C. An D. ứ (no article) Mark the letter A, B, C, or D on your answer sheet to indicate the word(s) CLOSEST in meaning to the underlined word(s) in each of the following questions. Question 20. When our rent increased from $100 to $200 a month, we protested against such a tremendous increase. A. light B. huge C. tiring D. difficult Question 21. The government decided to pull down the old building so that a hospital could be built. A. renovate B. purchase C. maintain D. demolish Mark the letter A, B, C, or D on your answer sheet to indicate the word(s) OPPOSITE in meaning to the underlined word(s) in each of the following questions. Question 22. It was necessary to divide the movie ‘Roots’ into five parts in order to show it on television. A. adapt B. merge C. segment D. transact Question 23. That is a well-behaved girl whose behaviour has nothing to complain about. A. good behavior B. behaving improperly C. behaving nice D. behaving cleverly Mark the letter A, B, C, or D on your answer sheet to indicate the option that best completes each of the following exchanges. Question 24. Peter’s teacher is commenting on his essay. -Teacher: “Peter, you’ve written a much better essay this time.” - Peter: “ ” A. Writing? Why? B. Thank you. It’s really encouraging. C. You’re welcome. D. What did you say? I’m so shy. Question 25. Linda and Daisy are talking about the environment. Linda: “Our environment is getting more and more polluted. Do you think so?” Daisy: “ . It’s really worrying.” A. I’ll think about that. B. I don’t agree. C. I don’t think so. D. I can’t agree more. Read the following passage and mark the letter A, B, C, or D on your answer sheet to indicate the correct word or phrase that best fits each of the numbered blanks from 26 to 30. The story of gold is an adventure involving kings, queens, pirates, explorers, conquerors, and the native peoples they conquered. Throughout history, gold has woven a magic spell over those it touched. Gold is beautiful and rare; a soft shiny metal that can be moulded into many (26)___ __. It has been used for money, jewelry, and to decorate special buildings such as palaces and places of worship. (27)__ __ the precious metal was discovered, prospectors rushed to mine it, starting new cities and countries as they went. Gold and the people who love it have helped shape the world we live (28)___ __ today. Gold is one of many elements, or substances that cannot be changed by normal chemical means, that are found in the Earth’s crust. Gold has a warm, sunny colour and (29)___ __ it does not react with air, water, and most chemicals, its shine never fades. In its natural (30)___ __, gold is soft and easily shaped. When heated to 1,062 Celsius it melts and can be poured into moulds to form coins, gold bars, and other objects. Stories have been told, P 156
  18. movies made and legends bom about the discovery of the world’s great gold deposits. It is a saga of dreams, greed, ambition and exploration. Question 26. A. formats B. outlines C. shapes D. lines Question 27. A. Whoever B. However C. Forever D. Wherever Question 28. A. at B. in C. for D. on Question 29. A. yet B. despite C. because D. so Question 30. A. estate B. stage C. state D. position Read the following passage and mark the letter A, B, C, or D on your answer sheet to indicate the correct answer to each of the questions from 31 to 35. The Hindu culture celebrates marriage as a pure and pristine rite enabling two individuals start their journey of life together. It puts emphasis on the values of happiness, harmony and growth and could be traced back from the Vedic times. Months before the wedding ceremony, an engagement is held which is called “magni”. The couple is blessed here with gifts, jewelry and clothes. Another important ritual is the “mehendi” which is a paste made from the leaves of henna plant. It is the traditional art of adorning the hands and the feet of the bride with mehendi and the name of the groom is also hidden in the design. On the day of marriage, the couple exchanges garlands as a gesture of acceptance of one another and a pledge to respect one another as partners which is known as “jaimala”. This is followed by “kanyadaan”, where the father of the bride places her hand in the groom’s hand requesting him to accept her as an equal partner. Another ritual is the “havan” in which the couple invokes Agni, the god of Fire, to witness their commitment to each other. Crushed sandalwood, herbs, sugar rice and oil are offered to the ceremonial fire. The “gath bandhan” takes place where scarves of the bride and groom are tied together symbolizing their eternal bond. This signifies their pledge before God to love each other and remain loyal. The couple then takes four “mangal pheras” or walk around the ceremonial fire, representing four goals in life: “Dharma”, religious and moral duties; “Artha”, prosperity; “Kama” earthly pleasures; “Moksha”, spiritual salvation. The couple also takes seven steps together to begin their journey, called the “saptapardi”. Then the ritual of “sindoor” takes place where the groom applies a small dot of vermilion, a red powder to the bride’s forehead and welcomes her as his partner for life. This signifies the completion of the marriage. The parents of the bride and the groom then give their blessings, “ashirwad” to the newly wed couple as they touch the feet of their parents. Question 31. What might be the most suitable title for this reading passage? A. The Hindu culture B. The Hindu religion C. The Hindu wedding D. The Hindu tradition Question 32. What do the Hindu people think about marriage? A. It is entirely a spiritual traditional ritual allowing two individuals to live together. B. It is a belief in the growth of a family newly formed by two individuals. C. It is a wish of happiness and harmony to come to two individuals. D. It is just a living-together announcement of two individuals. Question 33. What can the word “adorning” be best replaced by? A. decorating B. painting C. repairing D. dying Question 34. Why does the couple exchange garlands? A. to do a ritual B. to show both their acceptance of and the swear to respect the partner C. to express their acceptance of one another and a wish of happiness D. to express their vow to respect each other for the whole life Question 35. When is the wedding ceremony completed? A. When the parents of the bride and the groom give their blessings to the couple. B. When the couple touches their parents’ feet. P 157
  19. C. When the couple makes seven steps together. D. When the groom applies a small dot of vermillion of the bride’s forehead. Read the following passage and mark the letter A, B, C, or D on your answer sheet to indicate the correct answer to each of the questions from 36 to 42. The concept of urban agriculture may conjure up images of rooftop, backyard or community gardens scattered among downtown city streets and surrounding neighborhoods. But in the Seattle area, and within and beyond the Puget Sound region, it means a great deal more. “Urban agriculture doesn’t necessarily equate to production that occurs only in a metropolitan urban area, “says Jason Niebler, who directs the Sustainable Agriculture Education (SAgE) Initiative at Seattle Central Community College. “It means we are providing for growing population food needs from surrounding rural landscapes, as well as from the core urban landscape.” Picture a series of concentric circles, with an urban core that produces some food at varying capacities, surrounded by a series of outlying rings of small farms that become increasingly more rural with distance. The hope is that such land use planning, from the inner core to the outer rings, will encourage local ecologically sound sustainable food production. This, in turn, will create local jobs and decrease reliance on distant food products that originate from petroleum-intensive large scale farms. That’s the idea behind SAgE, believed to be the nation’s first metropolitan-based community college sustainable agriculture program that emphasizes farming practices across diverse landscape types from urban centers to surrounding rural environs. “It’s small scale agriculture with an urban focus,” Niebler says. “Any urban population, large or small, can practice sustainable agriculture, improve food security and protect the environment, which ultimately results in resilient food systems and communities.” SAgE is a part of National Science Foundation’s Advanced Technological Education (ATE) Program, which is providing the project with $157,375 over two years. ATE’s goal is to support projects that strengthen the skills of technicians who work in industries regarded as vital to the national’s prosperity and security. The support largely goes to community colleges that work in partnership with universities, secondary schools, businesses and industries, as well as government agencies, which design and implement model workforce initiatives. The SAgE project focuses on the environmental, socioeconomic, political and cultural issues related to sustainable food systems, within Puget Sound watersheds through student and community education and research, and technological innovation. The curriculum offers courses that cover such issues as agricultural ecology, urban food systems, food politics and ethics, soil science, sustainable food production and technology, the integration of food and forests, and career opportunities. “We’ve created a curriculum that is fundamental in nature, addressing the principles of sustainable agriculture and what a food system is - how it functions both locally and globally,” Niebler says. “These courses are challenging, robust and inspirational. One of the really wonderful things about them is that we offer service learning opportunities, where students volunteer a portion of their time to working with local partner organizations. They can do a research project, or a service learning option. The ideal would be to prompt students into careers that involve sustainable practices in an urban agriculture setting.” (‘‘Promoting Sustainable Agriculture ” by Mariene Cimons) Question 36. It is stated in the passage that Jason Niebler ___. A. preserves the core urban landscape B. provides food for Seattle’s population C. studies at Seattle Central Community College D. directs the SAgE Initiative Question 37. The word “concentric” in paragraph 2 is closest in meaning to ___. A. coming from different places B. having the same size C. going in different directions D. having the same center P 158
  20. Question 38. Which of the following is supposed to be an outcome of the SAgE’s new land use planning? A. dependence on distant food products B. increased food production in large scale farms C. employment opportunities for local residents D. modernized farming practices in rural environs Question 39. The curriculum of SagE at Seattle Central Community College offers courses covering the following EXCEPT ___. A. agricultural ecology B. career opportunities C. urban system development D. integration of food and forests Question 40. In Niebler’s opinion, the courses offered by the SAgE project are ___. A. functional but impractical B. robust but unpromising C. challenging and costly D. hard but encouraging Question 41. The word “them” in paragraph 6 refer to ___. A. courses B. opportunities C. principles D. students Question 42. Which of the following is NOT true according to the passage? A. The curriculum that the SAgE project designs is fundamental in nature. B. The SAgE project alone will offer student sufficient jobs in urban agriculture. C. ATE helps to improve the skill of technicians in the nation major industries. D. Resilient food systems can be attributed to sustainable agricultural practices. Mark the letter A, B, C, or D on your answer sheet to indicate the underlined part that needs correction in each of the following questions. Question 43: (A) On the table (B) is (C) hundreds of books written (D) in English Question 44: According to (A) most medical experts, massage (B) relieves pain and anxiety, eases depression and (C) speeding up recovery from (D) illnesses Question 45: We have conducted (A) exhausting research (B)into the effects of smartphones on students’ behaviour (C) and their (D) academic performance. Mark the letter A, B, C, or D on your answer sheet to indicate the sentence that is closest in meaning to each of the following questions. Question 46: If you practise harder, you will have better results. A. The harder you practice, the best results you will have. B. The more hardly you practice, the better results you will have. C. The hardest you practice, the most results you will have. D. The harder you practice, the better results you will have. Question 47: “You had better see a doctor if the sore throat does not clear up.” she said to me. A. She reminded me ofseeing a doctor ifthe sore throat did not clear up. B. She ordered me to see a doctor ifthe sore throat did not clear up. C. She insisted that I see a doctor unless the sore throat did not Clear up. D. She suggested that I see a doctor if the sore throat did not clear up. Question 48: I'm sure Luisa was very disappointed when she failed the exam. A. Luisa must be very disappointed when she failed the exam. B. Luisa must have been very disappointed when she failed the exam. C. Luisa may be very disappointed when she failed the exam. D. Luisa could have been very disappointed when she failed the exam. Mark the letter A, B, C, or D on your answer sheet to indicate the sentence that best combines each pair of sentences in the following questions. Question 49. The basketball team knew they lost the match. They soon started to blame each other. A. Hardly had the basketball team known they lost the match when they started to blame each other. P 159
  21. B. Not only did the basketball team lose the match but they blamed each other as well. C. No sooner had the basketball team started to blame each other than they knew they lost the match. D. As soon as they blamed each other, the basketball team knew they lost the match. Question 50. He wasn’t wearing a seat-belt. He was injured. A. If he hadn’t been wearing a seat-belt, he wouldn’t have been injured. B. If he had been wearing a seat-belt, he would have been injured. C. If he had been wearing a seat-belt, he wouldn’t be injured. D. If he had been wearing a seat-belt, he wouldn’t have been injured. Number 29: Mark the letter A, B, C, or D on your answer sheet to indicate the word whose underlined part differs from the other three in pronunciation in each of the following questions. Question 1. A. workshop B. worm C. worry D. wordless Question 2. A. borrowed B. helped C. dismissed D. booked Mark the letter A, B, C or D on your answer sheet to indicate the word that differs from the rest in the position of the main stress in each of the following questions. Question 3. A. dissolve B. household C. confide D. approach Question 4. A. determine B. argument C. counterpart D. marvelous Mark the letter A, B, C or D on your answer sheet to indicate the correct answer to each of the following questions. Question 5. Let's begin our discussion now, ? A. shall we B. will we C. don't we D. won’t we Question 6. John and Karen persuaded me the conference. A. attending B. attended C. to attend D. to have attended Question 7. If she a car, she would go out in the evening. A. have B. has C. has had D. had Question 8. By the time the boss comes back from England, the work . A. will have been finishing B. will be finishing C. will have been finished D. will be finished Question 9. Thirty-one people have been injured in___incidents throughout the day. A. violent B. violence C. violently D. violate Question 10. what chemical is this? It's___a horrible smell. A. giving over B. giving off C. giving down D. giving up Question 11. Mary will have finished all her work ___ A. as soon as her boss returned B. until her boss will return C. by the time her boss returns D. when her boss will return Question 12. The patients with the new drug showed better signs of recovery than those receiving conventional medicine. A. treated B. having treated C. treating D. who treated Question 13. It is probably impossible for life to ever exist on Venus___its intense surface heat. A. because B. although C. despite D. due to Question 14. If you cannot improved sales figures this month, you will be___sacked. A. promise B. conduct C. regulate D. deliver Question 15. We were expecting beautiful weather at the beach, but it was so cold and rainy that, ___ getting a suntan, I caught a cold. A. compared to B. just as C. in case of D. instead of Question 16. I think Trump can easily win the election. He is a man of the___. A. community B. society C. country D. people Question 17. Overcrowded slums are often considered as breeding ground___crime. A. of B. to C. for D. towards P 160
  22. Question 18. After a momentary of concentration, Simon managed to regain his focus and completed the test. A. failure B. lapse C. fault D. error Question 19. Did you read book I lent you last week? A. a B. the C. an D. no article Mark the letter A, B, C, or D on your answer sheet to indicate the underlined part that needs correction in each of the following questions. Question 20: The number (A)of students (B)attending universities to study (C)economics (D)have increased steadily in the last few years. Question 21: Though (A) formally close friends, they (B) have now been estranged from (C) each other due to some (D) regrettable misunderstandings. Question 22: The (A)happy, anxiety and worries were (B)all mixed up and (C)made me unable to sleep or eat for (D)days. Mark the letter A, B, C or D on your answer sheet to indicate the sentence that is closest in meaning to each of the following questions. Question 23. It would have been better if he had told us his new address. A. He might have told us his new address. B. He should have told us his new address. C. He shouldn’t have told us his new address. D. It doesn’t matter that he didn’t tell us his new address. Question 24. She knows more about it than I do. A. I know as much about it as she does. B. She knows as much about it as I do. C. I don’t know as much about it as she does. D. She doesn’t know as much about it as I do. Question 25. “Why don’t you participate in the volunteer work in summer?”, said Sophie. A. Sophie suggested me to participate in the volunteer work in summer. B. Sophie asked me why not participate in the volunteer work in summer. C. Sophie suggested my participating in the volunteer work in summer. D. Sophie made me participate in the volunteer work in summer. Mark the letter A, B, C or D on your answer sheet to indicate the sentence that best combines each pair of sentences in the following questions. Question 26: He didn’t take his father’s advice. That’s why he is out of work. A. If he had taken his father’s advice, he would not have been out of work. B. If he took his father’s advice, he would not be out of work. C. If he had taken his father’s advice, he would not be out of work. D. If he takes his father’s advice, he will not be out of work. Question 27: His wife gave birth to their first child. He understood what true responsibility meant. A. Never has he understood true responsibility before he became a parent himself. B. Were his first child not to be born, he wouldn’t understand true responsibility. C. Hardly had he understood true responsibility when their first child was born. C. Not until he became a parent did he understand what true responsibility meant. Mark the letter A, B, C or D on your answer sheet to indicate the most suitable response to complete each of the following exchanges. Question 28. - Laura: “What a lovely cat you have!” - Maria: “___.” A. Of course not, it’s not costly B. Thank you. Thank you. My father bought it for me C. I think so D. No problem Question 29. - Janet: "Do you feel like going to the cinema this evening?" - Susan: “___’’ P 161
  23. A. I don't agree, I'm afraid B. You’re welcome C. That would be great D. I feel very bored Mark the letter A, B, C, or D on your answer sheet to indicate the word or phrase that is CLOSEST in meaning to the underlined part in each of the following questions. Question 30. she got up late and rushed to the bus stop. A. came into B. went leisurely C. dropped by D. went quickly Question 31. School uniform is compulsory in most of Vietnamese schools. A. divided B. paid C. required D. depended Mark the letter A, B, C, or D on your answer sheet to indicate the word or phrase that is OPPOSITE in meaning to the underlined part in each of the following questions. Question 32. I must have a watch since punctuality is imperative in my new job. A. being courteous B. being cheerful C. being efficient D. being late Question 33. My sister lives in Alaska, so we can only see each other once in a blue moon. A. occasionally B. rarely C. at night D. frequently Read the following passage and mark the letter A, B, C or D on your answer sheet to indicate the correct word for each of the blanks from 34 to 38. VIRTUAL DOCTORS Clare Harrison rarely falls ill and hates going to the doctors when she does. So when she recently (34) ___out in a painful rash down one side of her body she emailed her symptoms, (35) ___also included a (36) ___fever, to e-doc, the internet medical service. Two hours later she was diagnosed as having shingles (Herpes Zoster) by her online doctor, who prescribed a special cleansing solution for the rash and analgesics to help relieve the pain. Health advice is now the second most popular topic that people search for on the internet, and online medical consultation is big business. Sites vary enormously in what they offer, with services ranging from the equivalent of a medical agony aunt to a live chat with a doctor via email. They are clearly (37) ___a demand from people who are too busy or, in some cases, too embarrassed to discuss their medical (38) ___with their general practitioners. Question 34. A. worked B. passed C. came D. ran Question 35. A. whose B. which C. who D. that Question 36. A. small B. weak C. mild D. calm Question 37. A. serving B. meeting C. creating D. establishing Question 38. A. harm B. story C. hardship D. complaint Read the following passage and mark the letter A, B, C, or D on your answer sheet to indicate the correct answer to each of the questions from 39 to 43. Dolphins are one of the most intelligent species on the planet, which makes them a very interesting animal to scientists. In their natural habitats, dolphins use various vocalization techniques. They whistle and squeak to recognize members of their pod, identify and protect their young, and call out warnings of danger. They also make clicking sounds used for echolocation to find food and obstacles in dark and murky waters. Amazingly, the whistling sound that the bottlenose dolphin makes has been found to have a similar pattern to human language. They always make conversational sounds when they greet each other. If you listen to dolphins' squeaks and squeals, it will sound like they are having a conversation. Dolphins usually use both sound and body language to communicate with each other. It is through gesture and body language, however, that most of their communication with humans comes. Dolphins can be trained to perform complicated tricks. This suggests they have a high level of intelligence and communication capacity. If they work for a long time with a trainer, they are able to recognize and understand human commands. A lot of dolphin communication has been studied using dolphins in captive environments. These studies have been criticized because some marine biologists believe that dolphins living in aquariums or research centers cannot be considered "normal." Even so, most believe that P 162
  24. studying dolphin communication in captivity is useful for beginning to understand the complexity of dolphin communication. After all, dolphins are one of the most intelligent animals. Their ability to communicate is impressive and worthy of study. Question 39. What would be the most suitable title for the passage? A. Communications in dolphins B. Intelligent mammals in captivity C. Dolphins' social tendencies with humans D. Complex patterns in dolphin life Question 40. In line 2, the word vocalization is closest in meaning to___. A. making gestures B. creating words C. producing sounds D. closing eyes Question 41. According to paragraph 4, what do some marine biologists think about captive dolphins? A. Their relationships are too personal with marine biologists. B. They can't be considered accurate subjects for biological studies. C. They can communicate exactly the same as humans do. D. They easily demonstrate how dolphins act in the wild. Question 42. All of the following are true about dolphin communication EXCEPT___ A. dolphins' squeals and squeaks sound conversational to the human ear B. dolphins' clicking sounds are sometimes used to greet humans C. communication using sounds and gestures occurs between dolphins D. body language and gestures are used for communication with humans Question 43. In line 17, the word “most” refers to ___ A. the majority of marine biologists B. a few bottlenose dolphins C. a lot of bottlenose dolphins D. the minority of marine biologists Read the following passage and mark the letter A, B, C, or D on your answer sheet to indicate the correct answer to each of the questions from 44 to 50. What does it take to graduate from university with a First? Mark McArdle, first-class degree holder from the University of Lancaster, tells how he did it. Don’t spend too much time at the student lounge, do turn up for most lectures and tutorials and do submit all coursework - eventually. That, I was told by a PhD student during freshers’ week, was all I needed to do to get a 2:2. For a 2:1, I’d require a better attendance record and have to work harder, but not at the expense of being cut off from civilization. And for a First I would have to become some sort of social outcast, go to even lecture and tutorial (scribbling notes madly), spend every waking moment immersed in academic books, and be among the last to be thrown out of the university library at 10 pm closing time. Well, I did not give up my life for study. I didn't attend every lecture and tutorial. I didn’t write down every word spoken in lectures. I didn’t get 80% or more in every essay, project, test or exam. I was usually behind with my reading and occasionally mystified by the syllabus. Sometimes I couldn't be bothered to go to university and stayed at home instead. But I always knew where I was what I had to do and what not to bother with. And I always worked hard on the things that counted: assignments and exams. Getting a degree is about learning, but it isn’t just about learning biology, history, English or whatever. It’s about understanding what you need to succeed - what, in fact, the university wants from you and what you will get in return. You have to have a feel for the education market and really sell your inspirations, what does the lecturer want? what is the essay marker searching for? Some students try to offer something not wanted. Others want to give very little - they steal the thoughts of others and submit them as their own. But they all want to be rewarded. Exchange, but don’t steal, and you’ll get a degree. I could guarantee every book on my reading list was out on long loan from the university library within five seconds of the list being issued. This was worrying at first, but I quickly P 163
  25. learned that it was impossible to read all of the books on an average reading list anyway. I sought shortcuts. Collections of selected readings or journal articles were excellent sources that often saved me the bother of reading the original texts. References in books dragged me all over the place but with all the courses I had to do, there wasn't enough time to be dragged too far. I would flick through the book, read the introduction, note any summaries, look at diagrams, skim the index, and read any conclusions. I plucked out what was needed and made my escape. I revised by discarding subject areas I could not face revising, reading, compiling notes, and then condensing them onto one or two sheets of A4 for each subject area. Leading up to the exam, I would concentrate on just the condensed notes and rely on my memory to drag out the detail behind them when the time came. I didn’t practice writing exam questions, although it was recommended. I prefer to be spontaneous and open-minded. I don't want pre-formed conclusions filling my mind. And nor should you; there is no secret to getting a First - this is just an account of how I got my First. Be a happy student by striking the right balance between working and enjoying yourself. Take what you do seriously and do your best. And, no matter what you do don’t forget to appreciate every day of your university studies: it is one of the greatest periods of your life. Glossary: University degree classifications in the UK: - First class - Upper second (2:1) - Lower second (2:2) - Third class (3) Question 44. What is the main purpose of the passage? A. To describe learning experience of an excellent student B. To emphasize the importance of higher education C. To suggest ways to deal with assignments at university D. To point out challenges of studying at university Question 45. The PhD students who spoke to the writer___. A. exaggerate the need to work hard B. succeeded in scaring the writer C. was uncertain how to help the writer D. thought the writer would get a First Question 46. In the third paragraph, the writer warns against___. A. offering money to academic staff B. trying to second-guess lectures C. expecting to be rewarded D. plagiarism in essays and exams Question 47. The word “immersed” in paragraph 1 is closest in meaning to___. A. ploughed B. absorbed C. sunk D. dipped Question 48. Why did the writer not practice writing exam questions? A. He thought the practice was rather boring. B. He wanted to answer exam questions critically. C. He was advised not to. D. He thought it might prejudice staff against him. Question 49. The word “flick through” in paragraph 4 is closest in meaning to___. A. search quickly B. skim C. borrow D. read carefully Question 50. All of the following are mentioned about factors that help the writer to succeed at university EXCEPT ___. A. an understanding of what was required B. regular attendance at lectures C. selective reading D. well-organizing revision Number 30 Mark the letter A, B, C, or D on your answer sheet to indicate the word whose underlined part differs from the other three in pronunciation in each of the following questions. Question 1. A. pressure B. future C. enthusiasm D. resume Question 2. A. played B. tried C. smiled D. wanted P 164
  26. Mark the letter A, B, C, or D on your answer sheet to indicate the word that differs from the other three in each position of primary stress in each of the following questions. Question 3. A. critical B. motivate C. horizon D. dominant Question 4. A. mistake B. unite C. wonder D. behave Mark the letter A, B, C, or D on your answer sheet to indicate the word or phrase that is CLOSET in meaning to the underlined part in each of the following questions. Question 5. The company will only employ competent engineers, so they want to see evidence of their work as well the references from previous employers. A. ambitious B. unqualified C. proficient D. inconvenient Question 6. The protesters were angry with the council’s plan to do away with a lovely old building and put a car park there instead. A. destroy B. replace C. remain D. keep Mark the letter A, B, C, or D on your answer sheet to indicate the word(s) that is OPPOSITE in meaning to the underlined word(s) in each of the following questions. Question 7. Unfortunately, the sunny intervals we were promised have been few and far between. A. unusual B. extraordinary C. rare D. frequent Question 8. On Saturday wearing uniform is optional, so I often choose T-shirt and shorts. A. acceptable B. compulsory C. uncomfortable D. voluntary Mark the letter A, B, C, or D on your answer sheet to indicate the underlined part that needs correction in each of the following questions. Question 9: A number of tourists (A) is going to (B) return the evaluation form distributed (C) by the travel agent (D) . Question 10: After teaching (A) English in Vietnam for (B) one year, Philip decided to buy (C) a house and spending (D) the rest of his life there. Question 11: The head teacher emphasized the need (A) for fairness(B) and equality between (C) the teachers (D) of the school. Mark the letter A, B, C, or D on your answer sheet to indicate the correct answer to each of the following questions. Question 12. His clothes are in a mess because he ___ the house all morning. A. will be painting B. has been painting C. will have painted D. had been painting Question 13. ___ proficiency in German would be of much help, it is not requirement for the advertised position. A. Despite B. Otherwise C. Regarding D. Although Question 14. Although David was ___ after a day’s work in the office, he tried to help his wife the household chores. A. exhaust B. exhaustively C. exhaustion D. exhausted Question 15. The old woman still recalls clearly ___ by her teacher when she was late on her first day at school. A. to be criticised B. to have criticised C. being criticised D. criticising Question 16. We have just visited disadvantaged children in an orphanage in Bac Ninh Province. A. located B. locating C. which locates D. to locate Question 17. If you plant these seeds in May, you ___ a garden full of flowers in October. A. would have B. had C. will have D. were having Question 18. Patients at highest ___ of complications can be detected based on artificial intelligence techniques. A. danger B. risk C. chance D. threat Question 19. If we didn't any measures to protect whales, they would disappear forever. A. use B. make C. take D. do P 165
  27. Question 20. The new management board is often described as the ___ force behind the company’s rapid expansions. A. driving B. leading C. rising D. heading Question 21. My uncle was ___ ill last month; however, fortunately, he is now making a slow but steady recovery. A. fatally B. heavily C. deeply D. seriously Question 22. I believe that judges should be independent ___ the government. A. to B. of C. with D. on Question 23. by the time we arrived at the restaurant. A. All of the food has been sold B. All of the food had been sold C. All of the food was sold D. All of the food was being sold Question 24. Apart from those three very cold weeks in January, it has been a very ___ winner. A. plain B. pale C. mild D. calm Question 25. Mary hardly ever cooks, ? A. did she B. didn't she C. does she D. doesn't she Question 26. In most developed countries, up to 50% of population enters higher education at some time in their lives. A. the B. a C. no article D. an Mark the letter A, B, C, or D on your answer sheet to indicate the sentence that is closet in meaning to each of the following questions. Question 27. “What have you done to your hair?” she said to her son. A. She asked her son what to do to his hair. B. She wanted her son to know what he had done to his hair. C. She wanted to know what did her son do to his hair. D. She asked her son what he had done to his hair. Question 28. Children tend to learn English better than adults. A. Adults tend to be the best at learning English. B. Children do not learn English as well as adults. C. Adults tend to learn English worse than children. D. Children tend to learn English more than adults. Question 29. The Smiths sent their first child to a boarding school, which was not a good idea. A. The Smiths shouldn’t have sent their first child to a boarding school. B. The Smiths could have well sent their first child to a boarding school. C. The Smiths didn’t need to have sent their first child to a boarding school. D. The Smiths can’t have sent their first child to a boarding school. Mark the letter A, B, C, or D on your answer sheet to indicate the sentence that best combines each pair of sentences in the following questions. Question 30: I didn't pay attention to the teacher. I failed to understand the lesson. A. Although I paid attention to the teacher, I failed to understand the lesson. B. I would have understood the lesson if I had failed to pay attention to the teache. C. I would have understood the lesson if I had paid attention to the teacher. D. Unless I failed to understand the lesson, I would pay attention to the teacher. Question 31: Peter told us about his leaving the school. He did it on his arrival at the meeting. A. Only after his leaving the school did Peter inform us of his arrival at the meeting. B. Not until Peter told us that he would leave the school did he arrive at the meeting. C. Hardly had Peter informed us about his leaving the school when he arrived at the meeting. D. No sooner had Peter arrived at the meeting than he told us about his leaving the school. Mark the letter A, B, C, or D on your answer sheet to indicate the most suitable response to complete each of the following questions. Question 32: Mai: "How fashionable a pair of trainers you have!” Nam: " ." P 166
  28. A. Do you want to know where I bought them? B. Thanks for your compliment. C. I know it's fashionable. D. Yes, of course. Question 33: Two friends are talking about the benefits of volunteering. Daisy: “As far as I know, doing charity work is a really helpful thing for everyone in the society.” Mark: “ .” A. That sounds great. B. I couldn't agree with you more. C.take part in this campaign. D. That's fine for me. Read the following passage and mark the letter A, B, C, or D on your answer sheet to indicate the correct word or phrase that best fits each of the numbered blanks from 34 to 38. ROSES According to fossil fuel records, roses are over 35 million years old and they were cultivated in China about 5,000 years ago. A Chinese emperor in the 6th century B.C. apparently had over 600 books on roses in his library, and oil was extracted from those grown in his gardens. (34) ___, only the highest members of society were allowed to use it. If anyone else was found with even a small amount, they were (35) ___ to death. Roses were also popular with the Romans, who used their petals as medicine, a source of perfume and as confetti at weddings. Cultivated rose were only introduced to Western Europe in the 18th century. Napoleon’s wife, Josephine, started a botanical garden near Paris, (36) ___ she collected all the known varieties of rose and encouraged the breeding of new ones. This led to the flowers becoming increasingly popular, and in Britain at that time roses became so (37) ___ that they were often used as currency in local markets. All roses in Europe used to be pink or white until the first red ones arrived from China 200 years ago. These now (38) ___ love and are the world’s most common cut flower. (Source: Face2face- Upper Intermediate – Student’s Book, by Chris Redston & Gillie Cunningham) Question 34. A. Furthermore B. However C. As a result D. Otherwise Question 35. A. sentenced B. killed C. let D. made Question 36. A. why B. that C. where D. who Question 37. A. worthless B. valuable C. prosperous D. priceless Question 38. A. symbolise B. symbolically C. symbolic D. symbol Read the following passage and mark the letter A, B, C, or D on your answer sheet to indicate the correct answer to each of the questions from 39 to 43. For many American university students, the weeklong spring break holiday means an endless party on a sunny beach in Florida or Mexico. In Panama City Beach, Florida, a city with a permanent population of around 36,000, more than half a million university students arrive during the month of March to play and party, making it the number one spring break destination in the United States. A weeklong drinking binge is not for anyone, however, and a growing number of American university students have found a way to make spring break matter. For them, joining or leading a group of volunteers to travel locally or internationally and work to show problems such as poverty, homelessness, or environmental damage makes spring break a unique learning experience that university students can feel good about. Students who participate in alternative spring break projects find them very rewarding. While most university students have to get their degrees before they can start helping people, student volunteers are able to help people now. On the other hand, the accommodations are far from glamorous. Students often sleep on the floor of a school or spend the week camping in tents. But students only pay around $250 for meals and transportation, which is much less than some of their peers spend to travel to more traditional spring break hotspots. P 167
  29. Alternative spring break trips appear to be growing in popularity at universities across the United States. Students cite a number of reason for participating. Some appreciate the opportunity to socialize and meet new friends. Others want to exercise their beliefs about people’s obligation to serve humanity and make the world a better place. Whatever their reason, these students have discovered something that gives them rich rewards along with a break from school work. Question 39. What is the passage mainly about? A. Students’ travelling preferences B. A traditional approach to spring breaks C. American students’ social life D. Students’ alternative spring breaks Question 40. How many university students travel to Panama Beach City every March for spring break? A. Around 10,000 B. Around 36,000 C. Around 500,000 D. Around 50,000 Question 41. The word “cite” in paragraph 2 probably means ___. A. listing B. getting C. avoiding D. inventing Question 42. The word “them” in paragraph 1 refers to ___. A. degrees B. people C. projects D. students Question 43. Which of the following is NOT mentioned as a problem that alternative spring break trips try to help solve? A. Environment damage B. Homelessness C. Poverty D. Overpopulation Read the following passage and mark the letter A, B, C, or D on your answer sheet to indicate the correct answer to each of the questions from 44 to 50. All over the country young people are entering a world of homelessness and poverty, according to a recent report by the housing group, Shelter. Nearly 150,000 young people aged between sixteen and twenty-five will become homeless this year, says Shelter. Some of the young homeless may sleep out in the open in such places as 'cardboard city' in London, where people of all ages sleep in the open air in their only homes - cardboard boxes. Others may find accommodation in shelters run by voluntary organizations or get a place in a hostel, which gives them board for up to ten weeks.’ But who are these people? Those who are seeking a roof over their heads are mostly not runaways but “throwaways” - people who have been thrown out of their homes or forced to leave because of parental divorce, an unsympathetic step-parent or one of many other reasons. Take the case of one sixteen-year-old schoolgirl, Alice. She did not come from a poor home and had just passed her exams with good results. The Shelter team met her in a hostel where she was doing her physics homework. It turned out that her parents had thrown her out of her home for no other reason that she wanted to do Science Advanced Level exams - which her parents refused her permission to do, saying that sciences were unladylike! Shelter says that the Government's laws do nothing to help these youngsters. Rising rents, the shortage of cheap housing and a cut in benefits for young people under the age of twenty-five are causing a national problem, according to Shelter. The recent changes in the benefit laws mean that someone aged between sixteen and twenty-five gets less than older people and they can only claim state help if they prove that they left home for a good reason. Shelter believes that because of the major cuts in benefits to young people, more and more are being forced to sleep on the streets. Shelter also points out that if you are homeless, you can't get a job because employers will not hire someone without a permanent address; and if you can't get a job, you are homeless because you don't have any money to pay for accommodation. It's an impossible situation. (Source: FCE success workbook) Question 44. Why was Alice turned out of her home? A. She refused to do her homework in the evening. B. She didn’t want to study for her Advanced Level Exams. C. She had not obtained desirable marks in her exams. D. Her parents didn’t argee with what she wanted to do. P 168
  30. Question 45. According to Shelter, once young people have been forced onto the streets, ___. A. they will never go back home again B. their benefits will be severely cut C. they will encourage their friends to do the same D. they will find it difficult to find work Question 46. The changes in the system of benefits mean that ___. A. young people do not receive as much money as those over twenty-five B. anyone under twenty-five and not living at home will receive help with food and accommodation C. the under twenty-fives can claim money only if they have left home D. young people cannot claim money unless they are under sixteen or over twenty-five Question 47. The word “benefits” in paragraph 4 are probably ___. A. extra wages for part-time workers B. financial support for those in need C. a law about distributing money D. gifts of food and clothing Question 48. What is the reading passage mainly about?’ A. The problem of unemployed people all over the world B. Timely actions from the governement to fight homelessness C. The plight of young, homless people D. An increasingly popular trend among youngsters Question 49. According to the passage, most young people become homeless because ___. A. circumstances make it possible for them to live at home B. they do not want to live with a divorced parent C. they have run away from home D. they have thrown away any chances of living at home by behaving badly Question 50. The word “Others” in paragraph 1 refers to ___. A. voluntary organizations B. young people C. people of all ages D. the young homeless elt. She was injured. A. If she hadn't been wearing a seat-belt, she wouldn't have been injured. B. If she had been wearing a seat-belt, she would have been injured. C. If she were wearing a seat-belt, she wouldn't be injured. D. If she had been wearing a seat-belt, she wouldn't have been injured. Question 50: Smoking is unhealthy. It also costs governments billions of dollars in health-care costs. A. Not only is smoking unhealthy but it also costs government billions of dollars in health-care costs. B. Whereas smoking is unhealthy, it costs government billions of dollars in health-care costs. C. If smoking is unhealthy, it will cost government billions of dollars in health-care costs. D. Smoking is so unhealthy that it costs government billions of dollars in health-care costs. P 169